Not signed in (Sign In)

Vanilla 1.1.9 is a product of Lussumo. More Information: Documentation, Community Support.

    • CommentAuthorquid
    • CommentTimeMar 1st 2013 edited
     

    Some hours ago I found a new answer in the Experimental Mathematics question http://mathoverflow.net/questions/12085/experimental-mathematics/123321#123321 [added: meanwhile also gone] to which I reacted quite harshly, downvote and flagged as spam. The reason for the latter is that this very same vixra-preprint got already mentioned I think at least three times on the site; an occassion I definitely could track down from just 10 days ago http://mathoverflow.net/questions/110944/what-does-the-numerically-verified-part-of-the-riemann-hypothesis-tell-about-prim/122295#122295 [deleted so not generally visible] then posted from a different (anonymous) acount with not much activity but trying to stir up in a in my opinion strange way some discussion around this same subject some 20 days earlier http://mathoverflow.net/questions/120017/why-mathematicians-do-not-accept-the-wu-sprung-semiclassical-model-as-a-solution [again deleted].

    The current posting is from a completely new account, which I however suspect to be not independent, without much evidence beyond doubting two independent mentions of that vixra-preprint.

    Yet so far I stayed somewhat alone in my harshness; which suggests it is perceived as overly harsh or the thing was just not yet generally seen. In either case, I thought asking here was a good idea.

  1.  

    Nah; you're fine quid. It looks spammy to me too.

    • CommentAuthorquid
    • CommentTimeMar 1st 2013 edited
     

    @Todd Trimble: thank you for the quick feedback! (Meanwhile, it is up to four flags.) [Added later it is deleted now.]